Med-Surg Student NCLEX questions

Pataasin ang iyong marka sa homework at exams ngayon gamit ang Quizwiz!

A priority nursing intervention for a patient with delirium is to: a) maintain patient safety b) eliminate precipitating factors c) increase physical activity d) implement reorientation

A a) Rationale: Maintaining patient safety is a priority nursing intervention while eliminating precipitating factors (Ch. 59, p. 1409). b) Rationale: Delirium is a reversible illness, most of the time therefore, by eliminating precipitating factors delirium is reversed. (Ch. 59, p. 1414) c) Rationale: Physical activity is important to prevent skin breakdown and adverse effects related to immobility (Ch. 59, p. 1416) d) Rationale: Reorientation is an intervention technique that the nurse can use, but it is not the most important (Ch. 59, p. 1416)

The pathologic processes that characterizes Multiple Sclerosis includes: Select all that apply. A. Chronic inflation B. Activated B-cells C. Demyelination D. Gliosis in the CNS E. Accelerated nerve impulses

A, C, D A Correct: antigen- antibody reaction within the CNS activates the inflammatory response C Correct: attacks on the myelin sheath of the neurons in the brain and spinal cord result to damage to the myelin sheath D Correct: as the inflammation subsides, glial scar tissue replaces the damaged tissue from inflammation, which leads to the formation of plaque

Focal injury can be minor to sever and can be localized to an area. Focal injury consists of what? Select all that apply a. Lacerations b. CFS leakage c. Hematomas d. Cranial nerve injuries

A,C,D

Which of the following patients is at the highest risk for developing osteoporosis? a) a 48-year-old Asian female who works in an outdoor garden b) a 59-year-old White male who walks 30 minutes three times per week c) a 66-year-old White female with a history of smoking d) a 71-year-old Black male with a body mass index of 20

C Page 1511

A 56 year-old female with a history of a myocardial infarction, hypertension and coronary artery disease presents to her primary care doctor with new onset bilateral dependent edema, fatigue and weight gain. What new diagnosis should the nurse anticipate that he will be providing patient education for? A. Right Sided Heart Failure B. Left Sided Heart Failure C. SVT D. Atrial Fibrillation

A. Right Sided Heart Failure

The nurse knows her teaching about postoperative care for a patient with a cast is effective when the patient states: a. "I will never elevate my arm above the level of my heart." b. "I will exercise my joints above and below my cast." c. "I will use a hanger to relieve an itch in my cast." d. "I will take the cast off myself to avoid being cut by the oscillating blade of the cast saw."

Correct answer: _B_ b) Instruct the patient to exercise joints above and below the cast. (Chapter 62, p. 1477)

A patient presents to the clinic complaining of intense throbbing on right side of her head. After diagnostic testing, the doctor has ruled out disease and injury and claims she is having a headache. What type of headache did the doctor most likely diagnose her with? a. Cluster headache b. Medication overuse headache c. Migraine headache d. Tension headache

Correct answer: ___C____ Rationale: for each answer options-with Chapter & page number a) Cluster headaches are focused in and around one eye, described as sharp, penetrating, or burning. Chapter 58 page 1370 b) Medication overuse headaches manifest as a worsening headache. Patient may complain of early awakening with decreased appetite, nausea, restlessness, decreased memory, and irritability. Chapter 58 page 1372-1373 c) Migraine headaches are described as intense throbbing, or pounding pain that involves one temple. The pain usually is unilateral but can be bilateral. Chapter 58 page 1370 d) Tension headaches are described as feeling a weight in or on the head or a band squeezing the head. Chapter 58 page 1370

1. A patient with bone marrow depression is undergoing a procedure, which of the following anesthetic agents would be contraindicated? a) isoflurane b) Propofol c) nitrous oxide d) ketamine

Correct answer: ____C___ Rationale: for each answer options-with Chapter & page number a) Isoflurane is not contraindicated in bone marrow depression, but is in CAD Chpt 18 pg. 325 b) Propofol is not contraindicated in bone marrow depression, but BP, and HR needs to be monitored during use Chpt 18 pg. 325 c) Nitrous oxide is to be avoided in patients with bone marrow depression, Chpt 18 pg. 325 d) Ketamine causes hallucinations and nightmares, but is not CI in bone marrow depression Chpt 18 pg. 325

A key aspect of teaching for the patient who was diagnosed with hypertrophic CMP includes which instructions? a) Avoid strenuous activity and dehydration. b) Do not elevate the feet to improve venous return to the hurt. c) Take vasodilators such as nitroglycerin to improve chest pain. d) Let the caregivers know that there is no need to learn CPR because of no risk for potential cardiac arrest.

A Rationale: a) Focus the teaching on helping patient avoid strenuous activity and dehydration. Any activity that cause an increase is systemic vascular resistance (thus increasing the obstruction to forward flow) is dangerous and should be avoided. Ch 36, p - 799. b) Rest and elevation of the feet will improve venous return to the hurt can manage chest pain and is beneficial. Ch 36, p - 799. c) Vasodilators such as nitroglycerin may worsen the chest pain by decreasing venous return and farther increasing obstruction of blood flow from the hurt. Ch 36, p - 799. d) Hypertrophic CMP is the most common cause of SCD. Need encourage caregivers to learn CPR because of the potential for cardiac arrest. Ch 36, p - 798, 800.

The nurse should teach the patient with osteoporosis the importance of (select all that apply): A. Participating in regular exercise including low impact weight-bearing activities B. Taking prescribed bisphosphonates on an empty stomach C. Avoiding sun exposure and wearing long clothing when going outside D. Increasing intake of calcium-rich foods

A, D Page 1512-1513

A 75 year-old male presents to the Emergency Department complaining of shortness of breath and new onset "fat ankles". After an initial exam which additional test(s) would likely be ordered to confirm a diagnosis of heart failure? (select all that apply) A. 12-lead EKG B. Chest X-Ray C. BNP or NT-proBNP D. CBC

A. 12-lead EKG B. Chest X-Ray C. BNP or NT-proBNP D. CBC

Mr. Smith is brought in to the ER with a head injury from a car accident. What manifestations could he present with that would make the nurse think he has elevated ICP? Select all that apply. A. Inability to move eye upward B. Bradycardia C. Headache D. Hypotension

A. Inability to move eye upward B. Bradycardia C. Headache

You have a patient with mitral valve stenosis. Which of the following are suspected findings with this diagnosis? Select all that apply. A. Low-pitched diastolic murmur. B. S3 and S4 sounds. C. Loud S1. D. Hemoptysis.

A. Low-pitched diastolic murmur. C. Loud S1. D. Hemoptysis

When should the nurse caring for a patient postop after vertebral disc surgery contact the HCP? A. Patient says he has a severe headache and the nurse notes there is drainage from the incision site. B. Patient complains of no relief and continued paresthesia after surgery. C. The patient's wife says his abdomen has been distended for several days and he has not had a bowel movement since before his surgery. D. Patient starts crying when the nurse suggests repositioning him in his bed.

A. Patient says he has a severe headache and the nurse notes there is drainage from the incision site.

The nurse has an 18-year-old patient with increased intracranial pressure caused by a head injury they sustained while playing sports. What is the best method for measuring ICP that the nurse would anticipate the physician to order? A. Ventriculostomy B. Fiberoptic catheter C. Air pouch/pneumatic technology D. Lumbar puncture

A. Ventriculostomy

What teaching should the nurse include when discharging a patient after a laminectomy? Select all that apply. A. Rock side to side while standing. B. Apply and remove the rigid orthosis brace by logrolling in bed. C. Invest in a mattress that is soft and comfortable to promote pressure point relief. D. Sitting and standing are encouraged, and prolonged amounts of time laying down and walking should be avoided. E. Apply and remove the rigid orthosis brace while sitting or standing.

ABDE A. Rock side to side while standing. B. Apply and remove the rigid orthosis brace by logrolling in bed. D. Sitting and standing are encouraged, and prolonged amounts of time laying down and walking should be avoided. E. Apply and remove the rigid orthosis brace while sitting or standing.

Which of the following patients are at risk for delirium? (select all that apply) a) A 45-yr old female presenting with a severe urinary tract infection (UTI). b) A 70-yr old male with a visual impairment c) A 27-yr old female in a coma after a motor vehicle accident d) A 32-yr old male undergoing alcohol withdrawal e) A 57-yr old obese male patient diagnosed with HF

ABDE a) Rationale: A UTI may result in dehydration which is a precipitating factor of delirium (Ch. 59, p. 1415) b) Rationale: Visual impairment precipitates delirium (Ch. 59, p. 1415) c) Rationale: Pt. is unconscious and non-responsive. (Ch. 56, p. 1317-1318) d) Rationale: alcohol withdrawal precipitates delirium (Ch. 59. P 1415). e) Rationale: HF is a chronic disease that may cause fluid and electrolyte imbalances resulting in delirium (Ch. 59, p. 1416).

A patient presents to the ED with wheezing during inspiration and expiration. During the assessment the nurse notices that the patient is no longer wheezing. What is the First priority for the nurse? a) Take a detailed health history b) Make a note in the chart that the patient is responding positively to treatment c) Contact the physician d) Measure the peak expiratory flow rate (PEFR)

Answer: C *pg 546

A 17 year old male patient is admitted with a head injury after a collision while playing basketball. After assessing the patient you notice he has developed clear nasal drainage, which action should the nurse take? a. When testing for CSF the ring around the blood will be black b. Check the drainage for glucose content c. Obtain a specimen of the fluid to send down to the lab for culture and sensitivity d. Have the patient title his head backwards to stop the drainage

B

When providing treatment and therapy education for a client with Rheumatoid Arthritis, the nurse is using teach back method to ensure proper education occurred. The nurse knows further education is needed when the patient makes which statement? a) "When using a heating pad to help relieve stiffness I will avoid using capsaicin and will remove the pad after 20 minutes." b) "I can expect to start seeing relief of symptoms within 1-2 weeks after starting drug therapy with Methotrexate." c) "When cleaning, it will be important for me to press water out of sponges and rags instead of wringing them out and rest between cleaning each room." d) "I will add daily walks or swimming into my exercise regimen, while maintaining the exercise program my physical therapist has planned."

B

A 32-year-old patient at the clinic was recently diagnosed with Rheumatoid Arthritis (RA). The nurse prepares to educate the client on some common clinical manifestation of RA that they may experience. The nurse knows to include the following: -Select all that apply a) Joint stiffness can be expected to worsen at the end of the day, closer to bedtime. b) Stiffness will occur mainly in the hands and feet, but wrists, elbows, knees and ankles can become affected as the disease progresses. c) The patient may experience asymmetrical joint stiffness. d) It may become difficult to grasp objects over time, especially smaller items.

B, D

Which patient is at greatest risk for acquiring pneumonia? a) 75-year-old who smokes cigarettes and has altered consciousness b) 45-year-old who uses IV drugs c) 65-year-old with cellulitis d) 30-year-old with an upper respiratory tract infection

Correct Answer: A a. Rationale: This patient has three risk factors for pneumonia. Being above 65-years-old, smoking and altered consciousness are all risk factors (Ch. 27, Pg. 501). b. Rationale: This patient only has one risk factor for pneumonia. Being under 45-years-old is not a risk factor, but IV drug use is a risk factor (Ch. 27, Pg. 501). c. Rationale: This patient only has one risk factor for pneumonia. Being 65-years-old or older is a risk factor, but having cellulitis is not a risk factor (Ch. 27, Pg. 501). d. Rationale: This patient has one risk factor for pneumonia. Being under 65-years-old is not a risk factor, but having an upper respiratory tract infection is (Ch. 27, Pg. 501).

The student nurse is providing teaching to the emergency department nurses concerning interventions to provide for a patient in respiratory failure. The nursing professor will intervene if the student states what? a) Augment coughing may benefit patients who are having secretions which are obstructing the airway and are too weak to cough. b) Too much hydration can increase the secretions of the patient with respiratory failure so it is important to limit fluid intake. c) Albuterol is a good option for acute bronchospasm relief if not contraindicated for the patient. d) Airway suctioning is appropriate if the patient is unable to expectorate secretions.

Correct Answer: B pg. 1617

When educating a patient about dietary changes to help control their hypertension, the nurse will suggest the following: (Select all that apply) a) Patient should consume whole-fat milk b) Daily sodium intake goal should be below 1500 mg/day. c) Healthy fats should be obtained from sources like fish oil. d) Calcium supplements will aid in reducing BP.

Correct Answer: B, C a) Patients should be advised to consume fat-free or low-fat milk products, not whole-fat. (Ch. 32, p. 688) b) 1500 mg/day is the recommended daily intake for sodium. This alone can drastically decrease blood pressure. (Ch. 32, P. 688) c) Omega-3 fatty acids found in certain fish oils can contribute to a reduction in BP and triglycerides. (Ch. 32, P. 688) d) Calcium supplements are not recommended to lower BP. (Ch. 32, P. 688)

1. A nurse attends an educational seminar on epistaxis. After the seminar the nurse understands which of the following patients is most at risk for epistaxis. A. A 67-year-old patient who is suffering from presbycusis B. A 22-year-old patient with an anatomical malformation of the sinuses C. A 62-year-old patient with hypertension and over use of decongestant nasal sprays D. A 30-year-old patient who is allergic to cats

Correct Answer: C A. Not correct: the patient is over 50 which increases their risk but presbycusis is general hearing loss form old age and is not related to nosebleeds. Ch26 P. 476 B. Not correct: the patient is under 30 but the anatomical malformation can increase the patient's risk, but this answer is not the most correct. Ch26 P. 476 C. Correct: The patient is over 50, does have allergies, and over uses decongestant spray which are all risk factors for epistaxis. The decongestant overuse can cause rebound inflammation resulting in an increased chance of having a nosebleed. Ch26 P. 476

All of the following patients have lung cancer. The nurse is most concerned about the prognosis of which patient? A. The patient coughing up blood-tinged sputum. B. The patient who complains of wheezing. C. The patient who complains of a hoarse voice. D. The patient who has lost weight and has jaundice.

Correct Answer: D. The patient who has lost weight and has jaundice. The patient with weight loss and jaundice presents with late-stage symptoms of lung cancer. Jaundice has developed due to the lung cancer metastasizing to the liver. Weight loss likely occurred to fatigue and anorexia, which are also late signs of lung cancer. Metastasis to the liver indicates a poor prognosis. The other answers are early signs of lung cancer.

A nurse is reviewing the use of an albuterol inhaler with a patient diagnosed with asthma. Which answers by the patient suggest the teaching has been successful? (Select all that applies) a) I can use this medication every day b) I will not exceed two puffs in 12 hours c) I will keep a diary to record medication use d) I should contact my health care provider if I use this medication more than two times per week.

Correct Answers: C & D *pg 554 & 550

A patient with acute osteomyelitis has been receiving a prolonged high-dose antibiotic therapy. Which comment, if made by the patient, indicates further teaching by the nurse is needed? (select all that apply) a) I should continue my IV antibiotics for at least 4 to 6 weeks b) I should not call my doctor when I experience bone pain c) I can stop my IV antibiotics once my symptoms improve d) I should call my doctor when I experience any hearing deficits, because this could be a toxic reaction to my antibiotic.

Correct answer/s: B, C a) Wrong: This comment does not indicate further teaching. Patients on antibiotic therapy for acute osteomyelitis should continue their antibiotic for at least 4-6 weeks or as long as 3-6 months (Bucher, 2017, pg 1498). b) Correct: This comment indicates need for further teaching. Nurses should encourage the patient to call their doctor about any bone pain, fever, swelling, or restricted limb movement so that treatment can be started (Bucher, 2017, pg 1499). c) Correct: This comment indicates need for further teaching. Nurses should stress the importance of continuing to take antibiotics after symptoms have improved (Bucher, 2017, pg 1499). d) Wrong: This comment does not indicate further teaching. Hearing deficits, impaired renal function, and neurotoxicity are all possible adverse reactions to prolonged antibiotic therapy (Bucher, 2017, pg 1499).

A registered nurse is caring for a patient with chronic osteomyelitis. What role can the registered nurse assign to the unlicensed assistive personnel? a) Teach patient and caregiver about antibiotic side effects b) Assist patient with passive range of motion of adjacent joints c) Assess wound for signs of infection d) Monitor color, temperature, capillary refill and pulses of the affected limb

Correct answer: B Rationale: for each answer options-with Chapter & page number a) Wrong: Only the RN can initially perform teaching, the LPN can reinforce teaching the patient. Teaching is not done by the UAP. (Bucher, 2017, pg 1498) b) Correct: UAP can assist patient with passive ROM of the adjacent joints and active ROM exercises of the unaffected limb. (Bucher, 2017, pg 1498) c) Wrong: Only the RN can assess for signs of infection. (Bucher, 2017, pg 1498) d) Correct: UAP can notify RN of patient complaints, but cannot monitor the patient for significant changes. This is commonly done by the LPN/LVN or RN themselves. (Bucher, 2017, pg 1498)

Diagnosis of SEID include all of the following EXCEPT a) Profound fatigue lasting at least 6 months b) Unrefreshing sleep c) Postexertional malaise: total exhaustion after even minor physical or mental exertion d) Previous diagnosis of fibromyalgia

Correct answer: D Rationale: for each answer options-with Chapter & page number a) Pg. 1548 Ch. 64 SEID is characterized by fatigue b) Pg. 1548 Ch. 64 SEID is characterized by unrefreshing sleep c) Pg. 1548 Ch. 64 SEID is characterized by total exhaustion after minor exertion d) Pg. 1546 Ch. 64 This is false because although fibromyalgia and SEID share common characteristics, no diagnosis of fibromyalgia is needed to be diagnosed with SEID

Which order would the nurse question? The patient has been recently diagnosed with osteoarthritis and is currently taking warfarin (Coumadin)? a) Capsaicin Cream (Zostrix) b) Ibuprofen (Motrin) c) Tylenol (acetaminophen) d) Doxycycline (Vibramycin)

Correct answer: ___B____ Rationale: for each answer options-with Chapter & page number a) Capsaicin cream depletes P from nerve endings, interrupting pain signals to the brain. Doesn't affect patients taking blood thinners. Chapter 64, Page 1520 b) Ibuprofen affects platelet aggregation, leading to a prolonged bleeding time. Patients taking both warfarin and a NSAID are at a higher risk for bleeding. Chapter 64, 1521 c) Patients with mild to moderate pain are recommended to start with Tylenol before moving to NSAIDS due to the risk of GI bleeding. Patients may receive up to 1000 mg every 6 hours. Chapter 64, 1523 d) Doxycycline is an antibiotic that decreases action of enzymes on cartilage degradation. Possible treatment alternative for patients with mild OA. Chapter 64, 1523

Upon diagnosis of SEID, the nurse should teach the patient a) To improve symptoms, exercise vigorously for at least three days a week. b) You should NOT use NSAIDs to treat your headaches, muscle and joint aches and fever. c) SEID will continue to worsen with time. d) You should eat a well-balanced diet, including fiber and fresh dark-colored fruits and vegetables for antioxidant action.

Correct answer: d a)Pg. 1548 Ch. 64 The nurse should teach the patient to perform small activities because major exertion will worsen the symptoms. b) Pg. 1548 Ch. 64. NSAIDs are not contraindicated for people with SEID. c) Pg. 1549 Ch. 64 SEID does not appear to progress and most patients recover or at least gradually improve over time. d) Pg. 1548-1549 Ch. 64 Eating a well-balanced diet will help improve symptoms of fatigue.

You are formulating a care plan for a 66-year-old patient who has advanced Parkinson's disease. His wife brought him in today concerned that he has lost 30 pounds in the past few months. Which of the following would be the most appropriate nursing intervention for the patient? a) Encourage serving three large meals a day that are high in fiber and fruit b) Recommend the use of low-dose polyethylene glycol daily c) Provide a diet plan that limits protein intake to evening meals d) Suggest serving her husband's food on a warmed plate, with his food cut into bite-sized pieces

D. A nursing diagnosis of imbalanced nutrition: less than body requirements related to inability to ingest food would be an appropriate nursing diagnosis for this patient (Ch. 38, p. 1391). a) While a diet high in fiber and fruit can help alleviate the common side effect of constipation that Parkinson's patients experience, it is recommended the patient eat 6 small meals a day as to be less exhausting than three large meals (Ch. 58, pg 1391). b) Polyethylene glycol is an anti-constipation medication. Parkinson's patients often experience constipation as a side effect of their medications anticholinergic properties and inadequate nutritional status. Some patients do use this medication for symptom relief but its regular administration is not key to this nursing diagnosis (Ch 58, pg. 1389, 1391). c) A diet plan that limits protein might be appropriate if the patient is taking Levodopa, but that is not the problem or diagnosis indicated in the question (Ch. 58, pg. 1391). d) Patients with Parkinson's can have a large amount of difficulty and frustration associated with eating due to hand tremors, dry mouth from medications and difficulty chewing and swallowing (dysphagia). Serving their food on a warmed plate with the pieces already cut, make the food more appealing and allow for easier feeding (Ch. 58, pg. 1389,1391).

The nurse is monitoring a cardiac patient when they recognize 5 premature ventricular contractions in a row. The QRS complexes are the same in shape, size, and direction. The patient's pulse is present and has preserved left ventricular function. After identifying precipitating causes, the patient is diagnosed with clinically stable monomorphic VT. The nurse may expect to administer which of the following drugs? a) IV procainamide b) IV Magnesium c) Atropine d) Phenytoin

A a) If the VT is monomorphic and the patient is clinically stable (pulse is present) and has preserved left ventricular function, IV procainamide, sotalol, or amiodarone is used. CH 35, Page 769-770. b) Polymorphic VT with a prolonged baseline QT interval is treated with IV Magnesium, isoproterenol, phenytoin, or anti-tachycardia pacing. CH 35, Page 770. c) An accelerated idioventricular rhythm (AIVR) may become symptomatic in which case atropine may be considered. CH 35, Page 770. d) Dopamine may be used as a temporary measure to increase HR and support BP until temporary pacing is started in patients with third-degree AV block. CH 35, Page 768.

When caring for a patient who has an exacerbation of Chronic Obstructive Pulmonary Disease (COPD), what medications would the nurse anticipate the provider to order? (Select all that apply) a) Corticosteroids b) Oxygen therapy c) Antihistamines d) Antibiotics e) Bronchodilators

A,B,D,E -many drugs are used for COPD, but histamines are not apart of the pathophysiology so anti-histamines do NOT treat COPD a) Rationale: Corticosteroids are effective for short-term use to treat exacerbation. Ch. 28 & Pg. 563, 564 b) Rationale: Administered in the hospital and may be titrated by ABG measurement. Ch. 28 & Pg. 564 c) Rationale: Antihistamines are NOT prescribed for exacerbation of COPD d) Rationale: The primary causes of COPD exacerbations are bacterial and viral. Antibiotics are used to treat exacerbations caused by bacterial infections. Ch. 28 & Pg. 564 e) Short-acting bronchodilators are a typical therapy for exacerbation and are preferred for those who are breathless. Ch. 28 & Pg. 564

A nurse is assessing a patient who has a possible diagnosis of Parkinson's Disease. During the patient's assessment, which of the following would be most important to their accurate diagnosis (Select all that apply). a) Festination b) Tremor that is prominent while performing activities c) Medication history d) Asymmetric onset of symptoms e) Double or blurred vision

A,C,D Diagnosis of Parkinson's Disease is complex and made using a patient's history and clinical features (Ch. 58 p. 1389). a) Festination is a shuffling gait and a very definitive feature of Parkinson's Disease (Ch. 38, p. 1389). b) A tremor is one of the 5 diagnostic features of Parkinson's Disease, and often the first sign. However, a tremor that is more prominent at rest is closely associated with Parkinson's Disease (Ch. 58, p. 1388). c) Some medications can cause parkinsonian symptoms. Medications such as: Haloperidol, Lithium, phenothiazine, reserpine and benzodiazepines. It is important that medication cause be ruled out before diagnosis is made (Ch. 58 P. 1387). d) According to the text, "Clinical diagnosis requires the presence of TRAP and asymmetric onset", therefore asymmetric onset would be a key feature in a patient's diagnosis of Parkinson's Disease (Ch. 58, p. 1389) e) Double vision is an early symptom of the similar degenerative neurological disease, Multiple Sclerosis, but not of Parkinson's Disease (Ch. 58, p. 1384)

A student nurse is taking care of a patient who has just been admitted to the ED with a possible diagnosis of respiratory failure. Which of the following diagnostic studies would the student nurse predict will be ordered for the patient? (select all that apply) a) Chest x-ray b) ABG analysis c) CT scan d) Blood and sputum cultures

Answers: A, B, C, D pg. 1614

Which of the following individuals are at high risk for developing lung cancer? Select all that apply. A. An 40-year-old male who has smoked cigarettes for 20 years. B. A 19-year-old male that works in a coal mine. C. A 31-year-old female who works as a X-ray technician. D. A 12-year-old female whose father smokes cigarettes inside the house. E. A 68-year-old male who has a history of TB and bronchiectasis.

Answer: A.B.C.D.E. a.Smoking is directly related to the development of lung cancer. b.Industrial agents such as coal dust, nickel, uranium, chromium, formaldehyde, and arsenic can increase the risk for lung cancer. c.Exposure to ionizing radiation (ex. X-rays) puts you at a higher risk for developing lung cancer. d.Exposure to secondhand smoke puts you at high risk for developing lung cancer. e.A person with a history of TB and bronchiectasis are at increased risk for developing lung cancer.

Which of the following is NOT an effective coping intervention for the nurse to teach to patients with Chronic Obstructive Pulmonary Disease? a) Exercise training b) Smoking Cessation counseling c) Texting and instant messaging d) Avoid cold or wet weather

Answer: D-avoid cold or wet weather a) Rationale: Exercise training leads to energy conservation, which is an important part of COPD rehabilitation. Ch. 28 & Pg. 572 b) Rationale: Counseling the patient in smoking cessation is vital, because it is the only way to slow the progression of COPD. Ch. 28 & Pg. 572 c) Rationale: Sometimes patients with COPD have trouble speaking because of shortness of breath. These are good alternatives. Ch. 28 & Pg 573 d) Rationale: Patients frequently ask if moving to a warmer or drier climate will help. This is discouraged. Ch. 28 & Pg. 574

The nurse is monitoring a patient who came in with angina resulting in myocardial ischemia. The nurse monitors the 12-lead ECG and recognizes which of the following as signs that this potentially reversible stage of ischemia may be developing into myocardial infarction and must be treated immediately? (Select all that apply) a) The T wave is inverted. b) ST segment is elevated 2mm above the isoelectric line in two contiguous leads. c) A deep and wide Q wave that is wide (0.04) seconds in duration and about 30% of the height of the R wave. d) ST segment is depressed 2mm below the isoelectric line in two contiguous leads.

B,C a) ST segment elevation is not significant. Although it is 1mm or more above the isoelectric line, it is only seen in one of the leads. CH 35, Page 777. b) ST segment elevation is significant because it is 2mm above the isoelectric line in two contiguous leads. For the ST segment elevation to be significant it needs to be at least 1mm or more above the isoelectric line in two or more contiguous leads. CH 35, Page 777. c) A pathologic Q wave that develops during MI is wide (greater than 0.03 second in duration) and deep (greater than or equal to 25% of the height of the R wave). CH 35, Page 777. d) ST segment depression is seen in myocardial injury (NSTEMI). CH 35, Page 777.

A nurse has a patient who is experiencing angina. Which patient would the nurse monitor cautiously for side effects of hypotension and worsening chest pain when administering nitroglycerin? A. Patient with pericarditis B. Patient with aortic stenosis. C. Patient with mitral valve stenosis. D. Patient with atrial fibrillation.

B. Patient with aortic stenosis.

A patient with Multiple Sclerosis has acute exacerbations, what medication should the nurse expect to be most helpful to treat these exacerbations? A. Plasmapheresis B. Muscle relaxants C. Corticosteroids D. IV immunoglobulin G

C. Corticosteroids Correct: most helpful to treat acute exacerbations of MS. They reduce edema, and acute inflammation at the site of demyelination.

Which of the following are considered to be potentially ototoxic drugs? Select all that apply. A. Aspirin B. Loop diuretics C. Aminoglycosides D. Nonsteroidal anti-inflammatory drugs (NSAID's)

Correct answer/s: _A,B,C,D__(all of the above)____ Rationale: for each answer options-with Chapter & page number a) A high intake of aspirin has been known to cause tinnitus (Chapter 20, pg. 362) b,c,d) People taking diuretics, antibiotics, NSAID's and chemotherapy drugs should be monitored for hearing and balance problems. Hearing loss can be reversible if treatment is stopped with these drugs. (Chapter 20, pg. 362)

What patient education should be included upon discharge for a patient recently prescribed Labetalol for their hypertension? (Select all that apply) a) Instruct patient not to discontinue drug abruptly. b) Double up on doses when a dose is missed. c) The patient should measure their blood pressure before taking this drug. d) Erectile dysfunction is a possible side effect of this drug.

Correct Answer: A, C, D a) Beta blockers should not be discontinued abruptly. Doing so could cause rebound hypertension and potentially a hypertensive crisis. (Ch. 32, p. 699) b) Doses should not be doubled if a dose is missed. (Ch. 32, p. 697) c) Before taking beta blockers, patients should measure their blood pressure.(Ch. 32, p. 697) d) Some beta blockers cause sexual problems such as erectile dysfunction or decreased libido. (Ch. 32, p. 697)

Which of the following measures are important for the nurse to implement in order to prevent a patient at risk for aspiration from acquiring pneumonia? (Select all that apply) a) Maintain head of bed at 30 degrees b) Sit up for all meals c) Assess for gag reflex before giving food or fluids d) Reposition patient every two hours

Correct Answer: A,B,C a. Rationale: Elevating the head of the bed will decrease the risk for aspiration (Ch. 27, Pg. 505). b. Rationale: Sitting up will decrease the risk of aspirating food while swallowing (Ch. 27, Pg. 505). c) Rationale: The gag reflex is a defense mechanism for preventing food to go down the trachea. Assessing this before giving food or fluids will help prevent the aspiration of food (Ch. 27, Pg. 505) d) Rationale: This does not prevent aspiration. Frequently turning a patient will allow the lungs to expand and break up secretions (Ch. 27, Pg. 505).

What postoperative teachings are appropriate for a nurse to provide a patient undergoing a surgical cataract removal procedure? (select all that apply) a) Teach patient importance of coughing and deep breathing b) Inform patient that they may experience blurriness or scratchiness in operative eye c) Teach patient proper administration of antibiotic and corticosteroid eye drops d) Teach patient importance of independent ambulation

Correct answer(s): B, C Rationale: for each answer options-with Chapter & page number a) Rationale: (Ch. 21 p. 375) Patients should avoid activities that increase the intraocular pressure, such as bending or stooping, coughing, or lifting. b) Rationale: (Ch. 21 p. 375) After cataract surgery, the patient may have some scratchiness or blurriness in the operative eye. c) Rationale: (Ch. 21 p. 375) Postoperative medications for cataract removal surgery usually include antibiotic drops to prevent infection and corticosteroid drops to decrease the postoperative inflammatory response d) Rationale: (Ch. 21 p. 376) The nurse should suggest ways to mitigate fall risk by suggesting getting assistance with steps, removing area rugs and other potential obstacles, preparing meals for freezing before surgery, or obtaining audio books for diversion until visual acuity improves.

A patient with Acute Respiratory Distress Syndrome is at risk for the potential complication of barotrauma. Which of the following interventions will the nurse anticipate to avoid this complication? Select all that apply. A. Decrease tidal volume of 9 ml/kg to 6 ml/kg B. Maintain positive end-expiratory pressure at a constant 2 cmH2O C. Assess the patient for preexisting intracranial pressure elevation D. Maintain a gradual increase in PaCO2levels while assessing for patient tolerance

Correct answer/s: A, C, D Pg. 1623

The nurse is assessing a patient who came in the emergency room complaining of chest pain. Which of the following would alert the nurse to a possible myocardial infarction (Select all that apply) a) Patient states, "I have been taking antacids for indigestion but it hasn't been working." b) The patient's skin appears shiny and swollen, brown/brawny around ankles and lower legs. c) Crackles are heard upon auscultation of lungs d) Patient has a temperature of 100.4º F e) Patient describes very painful sensations around the substernal area when active but as soon as he sits down to rest the pain goes away.

Correct answer/s: A, C, D Rationale: a) Common locations for pain associated with myocardial infarction are the substernal or epigastric areas. Patients often attribute pain in the epigastric area as indigestion and try to treat it as such by taking antacids. (Chapter 33, pg 720) b) Shiny, swollen skin with brown/brawny color around the ankles and lower legs is indicative of chronic heart failure (Chap 34, pg 743). Skin changes for a myocardial infarction would most likely be cool, clammy and ashen in color (Chap 33, pg 720) c) Crackles may be heard during auscultation of lung sounds and could indicate left ventricular dysfunction related to an infarction in the left ventricle (Chap 33, pg 720). d) An elevated temperature/fever could be the result of a systemic inflammatory response due to heart cell death after a myocardial infarction (Chap 33, pg 720) e) Pain with activity that goes away during rest is more likely associated with chronic stable angina in which the pain is associated with a decreased supply of Oxygen or an increased demand for oxygen during activity. Chronic stable angina may indicate tissue ischemia but if it is treatable or reversible then it is not an infarction. However, prolonged tissue ischemia that is not treated or reversed can lead to infarction (Chap 33, pg 712 & 718).

The registered nurse is overseeing a patient with a pulmonary embolus. Which of the following are appropriate nursing interventions for this patient? Select all that apply. A. The nurse places the patient's head-of-the-bed to a semi-Fowler's position. B. The nurse tells the patient that they need to be on restricted bed rest for at least 7 days to prevent further growth or extension of thrombi. C. The nurse provides the patient with intravenous fluids to support adequate perfusion. D. The nurse provides patient education regarding "turn-cough-deep breath" methods.

Correct answer/s: A, C, D a) Rationale: Raising the head-of-the bed to a semi-Fowler's position is an appropriate intervention because it facilitates breathing (Lewis, Bucher, Heitkemper & Harding, 2017, p. 531). b) Rationale: Early ambulation for post-operative care is influential in minimizing the further growth or extension of a thrombi from upper or lower extremities, therefore restricted bed rest would be counterproductive for this particular patient (Lewis et al., 2017, p. 530). c) Rationale: A common drug therapy prescribed for patients with a pulmonary embolus are intravenous (IV) fluids in order to support adequate perfusion (Lewis et al., 2017, p. 530). d) Rationale: Patient education regarding the "turn-cough-deep breath" method allows for prevention or treatment of atelectasis associated with a pulmonary embolus (Lewis et al., 2017, p. 530).

The nurse's responsibilities when caring for a patient who presents to the emergency department with a Tension pneumothorax would expect to see: (Select all that apply) a. Profuse diaphoresis b. Heart rate of 140 beats per minute c. Tracheal deviation d. Absent breath sounds on the affected side

Correct answer/s: A. Profuse diaphoresis B. Heart rate of 140 bpm C. Tracheal Deviation D. Absent breath sounds on the affected side Rational: a. Profuse diaphoresis: Profuse sweating is a manifestation of a tension pneumothorax (Chapter 27 page 520). b. Heart rate of 140 beats per minute: A patient who is suffering from Tension Pneumothorax will have marked tachycardia. Tachycardia is defined as a heart rate of 100 beats per minute or greater (Chapter 29 page 598). c. Tracheal Deviation: The mediastinum shifts toward the unaffected side compressing the "good" lung, which further compromises oxygenation. This pressure ultimately causes the trachea to shift towards the good lung and become deviated (Chapter 27 page 520). d. Absent breath sounds on the affected side: As the pressure builds up in the good lung's pleural space, breath sounds will be either diminished or absent. (Chapter 27 page 520)

The nurse would expect which interventions for their patient who has acute angle-closure glaucoma? Select all that apply. A. β -Adrenergic blockers B. Cholinergic Agents (Miotics) C. Oral/IV hyperosmotic agents D. Argon laser trabeculoplasty (ALT)

Correct answer/s: B, C Rationales: a ) Incorrect - B-adrenergic blockers are drugs used as an initial treatment for chronic open-angle. (Ch 21 pg 380) b) Correct - Miotics are usually successful in immediately lowering the IOP. Acute angle-closure glaucoma is an ocular emergency that requires immediate intervention. (Ch 21 pg 380) c) Correct - Oral/IV hyperosmotic agents are also options for immediately lowering IOP which is the initial goal for acute angle-closure glaucoma. (Ch 21 pg 380) d) Incorrect - Argon laser trabeculoplasty (ALT) is a noninvasive option for chronic open-angle glaucoma to lower IOP when medications are not successful or when the patient cannot or will not use the drug therapy as recommended. (Ch 21 pg 380)

Which of the following symptoms are suspected in a patient experiencing Cushing's triad? (select all that apply) a) Increased heart rate b) Increased systolic blood pressure c) Irregular respiratory pattern d) Weight gain with a rounded face e) Increased intracranial pressure

Correct answer/s: B, C, E Rationale: a) Rationale: Bradycardia with a full and bounding pulse is a manifestation of Cushing's triad, not tachycardia. (Ch 56, p. 1318) b) Rationale: Increased systolic hypertension with a widening pulse pressure is a manifestation of Cushing's triad. (Ch 56, p. 1316 & 1318) c) Rationale: Irregular respiratory rate and pattern are manifestations of Cushing's triad, including patterns such as Cheyne-Stokes, central neurogenic hyperventilation, apneustic breathing, cluster breathing, and ataxic breathing. (Ch 56, p. 1318 & 1324) d) Rationale: Weight gain and a rounded face are manifestations or Cushing's syndrome, not Cushing's triad. (Ch 49, p. 1175) e) Rationale: Cushing's triad results from brainstem compression due to a severe increase in intracranial pressure and is a medical emergency that leads to death if left untreated.

Which of the following are characteristics of a properly functioning pleural drainage system? -Select all that applies a) Sudden cessation of tidaling within the water seal chamber indicates that proper pressure within the lung has been restored. b) The third chamber of the drainage system applies suction. c) Pleural drainage is used to collect fluid, air, and/or blood from the thoracic cavity. d) As the air leak resolves and the lung becomes properly expanded, bubbling within the second compartment ceases.

Correct answer/s: B,C,D_______ Rationale: for each answer options-with Chapter & page number a) "Investigate any sudden cessation of tidaling, since this may signify an occluded chest tube" Ch27, pg 523 b) "The third compartment, the suction control chamber, applies suction to the chest drainage system." Ch 27, Pg 523 c) "Insertion of a chest tube often requires attachment to a drainage device or chamber to collect fluid, air, and/or blood from the thoracic cavity." Ch 27, pg 522 d) "Intermittent bubbling during exhalation, coughing, sneezing (when the patient's intrathoracic pressure is increased) may be observed as long as there is air in the pleural space. Eventually, as the air leak resolves and the lung becomes more fully expanded, bubbling ceases." Ch 27, pg 522

A nurse is assessing a patient with Huntington's disease. Which of the following assessment data, if exhibited by the patient, would be of concern to the nurse and indicate that an abnormal event is occurring outside the expected disease progress of Huntington's disease? Select all that apply. A. The patient is having difficulty concentrating and needs to have instructions repeated. B. The patient is experiencing excessive and involuntary writhing, twisting movements in their face, limbs, and body. C. Crackles are heard in the lungs. D. The patient has begun to express a flat affect and has lost 5 pounds over the past few days.

Correct answer/s: C and D a) Rationale: This assessment data would be expected of an individual with Huntington's disease. There is no cure for Huntington's. As this disease progresses, cognitive function deteriorates and can involve variation in perception, memory, attention, and learning. Chapter 58, page 1396. b) Rationale: This assessment data would be expected of an individual with Huntington's disease. The excessive involuntary movements of writhing and twisting throughout the muscles of the body are referred to as chorea and are characteristic clinical manifestations of Huntington's disease. Chapter 58, page 1395 and 1396. c) Rationale: This assessment data is abnormal and should be very concerning to the nurse. Because psychomotor ability deteriorates with the disease progress, the ability to eat and speak are impaired. This puts the patient at risk for aspiration and pneumonia, which is the leading cause of death for patients with Huntington's disease. Chapter 58, page 1396. d) Rationale: While a flat affect may be expected with cognitive decline in Huntington's disease, an onset of behavioral changes accompanied by weight loss should indicate to the nurse that depression is compromising nutritional intake. Because of the choreic movements of Huntington's disease, as many as 4000 to 5000 calories a day may be needed to maintain body weight. On top of difficulty swallowing and holding the head still, depression and mental deterioration can compromise nutritional intake and put the patient at risk for a variety of complications. Chapter 58, page 1396.

Which patients are most likely to develop osteoarthritis? (Select all that applies) a) A 55-year-old male who is taking Indocin. b) A 65-year-old male who has a BMI of 30 c) A 45-year-old woman who has had multiple knee surgeries. d) A 30-year-old male who weight trains 4 days a week

Correct answer/s: _A, B, C______ Rationale: for each answer options-with Chapter & page number a) Drugs like indomethacin (Indocin) or corticosteroids can stimulate collagen digesting enzymes in joint synovium. Chapter 64, Table 64-1, 1518 b) After the age of 55 men hip OA is more common in men than women. Patients who are overweight are at a higher risk of developing OA. Chapter 64, 1518 c) After the age of 45 women have a higher risk of developing OA in the knee than men do. Multiple surgeries can put risk to the affected joint. Chapter, 1518 d) Weight training can prevent and strengthen affected joints in people who suffer from OA. Chapter 64, 1524

What medications can cause medication overuse headaches? Select all that apply? a. NSAIDs b. Topiramate c. Opioids d. Verapamil

Correct answer/s: _A, C___ Rationale: for each answer options-with Chapter & page number a) Patients with frequent headaches may overuse medications resulting in a headache that is worse than original headache. Medications include: acetaminophen, ASA, NSAIDs, butalbital, triptans, ergotamine, and opioids. Treatment involves abrupt withdrawal of offending drug except opioids which must be tapered. Chapter 58 page 1372-1373. b) Taken daily for migraine prevention. Must be used for 2-3 months to determine effectiveness. Patient must have adequate fluids to decrease risk of renal stone development. May cause seizures. Chapter 58 page 1372. c) Patients with frequent headaches may overuse medications resulting in a headache that is worse than original headache. Medications include: acetaminophen, ASA, NSAIDs, butalbital, triptans, ergotamine, and opioids. Treatment involves abrupt withdrawal of offending drug except opioids which must be tapered. Chapter 58 page 1372-1373. d) Verapamil is the first-choice drug in treating cluster headaches. Effects cardiac conduction. Chapter 58 page 1372

A nurse cares for a patient with a tracheostomy. The patient suddenly becomes tachypneic, has abnormal breath sounds, and starts using accessory muscles to breathe. The nurse is suspecting tracheostomy obstruction. The nurse should do what to manage the patient's airway? Select all that apply. A. Unlock and take out inner cannula and inspect for any obstruction, clean if necessary. B. Prepare the patient for a cricothyroidotomy. C. Offer the patient water to drink. D. Suction the tracheostomy.

Correct answer/s: _A, D______ A. Cleaning the inner cannula removes mucus from the inside of the tube to prevent airway obstruction B. This is a surgical procedure, similar to a tracheostomy, and would not help with the current tracheostomy obstruction- C. While fluids help thin out secretions, in an acute case this could further difficulty breathing if there is an occlusion, partial or complete, in the tracheostomy. This could also increase a risk for aspiration if there is a swallowing dysfunction- D. Suctioning at a pressure no greater than 125 mmHg for 10-15 seconds at a time can clear secretions from the airway and increase O2 saturation of the patient. -Chapter 26, Page 488

What initial assessments should the Registered Nurse perform on admission of the patient to the Post-Anesthesia Care Unit (PACU)? Select all that applies a) Airway patency b) Blood pressure c) Nausea, vomiting d) Height and weight

Correct answer/s: _A,B,C______ Rationale: a) Airway patency (as well as breathing and circulation) should be checked initially to make sure that oxygenation and ventilation are not affected by residual neuromuscular blockade such as opioids (p331). Airway obstruction is also commonly caused by blockage of air by the patient's tongue (p332). b) In the PACU, the most common cause of hypotension is fluid and blood loss, which may lead to hypovolemic shock. Hemorrhage is always a risk of surgery. Hypertension can also occur as a result of sympathetic nervous system stimulation from pain, anxiety, bladder distention, or respiratory distress (p337). c) Postoperative nausea and vomiting are the most common complications affecting as many as 80% of patients. Risk factors include younger age (<50 y/o), female gender, history of motion sickness, nonsmoking status, action of anesthetics or opioids, and duration and type of surgery (p341). d) Height and weight of the patient are not relevant initially in the PACU and therefore are not assessed by the RN at this time (p332).

A patient is prescribed a corticosteroid nasal spray as drug therapy in response to their allergic rhinitis. An intervention is needed if the patient makes which statement - (Select all that apply) a) I will use my nasal spray on a PRN basis. b) I will clear my nasal passage before using the spray. c) I will see the maximum benefits of the nasal after the first use. d) I will discontinue use if a nasal infection develops.

Correct answer/s: _A,C__ Rationale: for each answer options-with Chapter & page number a) Patients need to use corticosteroid nasal spray on a regular basis not PRN. - Chapter 26 Pg. 479 b) Patients must clear nasal passage before using the spray- Chapter 26 Pg. 479 c) The spray decreases inflammation and it may take several days or weeks to achieve maximum effects. - Chapter 26 Pg. 479 d) If a nasal infection develops it's important to discontinue use. In rare instances a side effect of nasal corticosteroid nasal spray is a localized fungal infection with Candida albicans. Chapter 26 Pg. 479

The nurse is providing preventative care to an older adult who is at risk for falls. Which implementations should the nurse teach the patient? (Select all that apply) a. Participate in moderate exercise b. Wear non-skid hard-soled footwear c. Remove scatter rugs d. Consume adequate Calcium and Vitamin D intake

Correct answer/s: __A, B, C, D__ a) Encourage older adults to participate in moderate exercise to help maintain muscle strength and balance. (Chapter 62, p. 1475) b) To reduce risk for falls, urge them to wear non-skid, hard-soled footwear (Chapter 62, p. 1475) c) Assess their living environment for safety risks (e.g., remove scatter rugs, ensure adequate lighting, maintain clear paths to the bathroom for nighttime use). (Chapter 62, p. 1475) d) Stress the importance of adequate calcium and vitamin D intake for bone health. (Chapter 62, p. 1475)

Which of the following would be important for the nurse to teach the patient regarding modifiable risk factors for Coronary Artery Disease? -Select all that apply a) Monitoring BP at home and adhering to blood pressure medication regimen b) Decreasing physical activity due to increased risk for cardiovascular event c) Changing eating patterns d) Monitor blood glucose levels regularly

Correct answer/s: __A, C & D__ Rationale: for each answer options-with Chapter & page number a) It is important for your patient to monitor their BP at home. Hypertension is considered the second major modifiable risk factor for the development of Coronary Artery Disease and can be treated with appropriate medications. If the patient did not monitor their Blood pressure at home, they may not be able to detect if their Blood Pressure medication is effective, thus not reducing their risk for a cardiovascular event. Found on page 705 in chapter 33 b) Decreasing physical activity contributes to an increased risk for the development for CAD. Physical activity helps with overall weight reduction and reduction of systolic Blood Pressure. Using the FITT (Frequency, intensity, type and time) formula to develop a workout program is recommended. Found on page 708 in Chapter 33 c) A change in diet with an emphasize in decreased saturated fats and cholesterol with an increase in complex carbohydrates and fiber has shown to promote reversal of Coronary Artery Disease and reduce heart events. Found on page 708 & 709 in Chapter 33 d) Diabetes is considered a modifiable risk factor with the control of blood glucose. An increase in glucose in the blood can lead to endothelial damage, promoting the formation of atherosclerotic plaque. That being said, it is vital for the patient to regularly check Blood Glucose levels, compensate with insulin/medications if necessary and pay attention to the foods they are eating. Found on page 708 in chapter 33

The patient has been diagnosed with oral cavity cancer what symptoms are the patient most likely to exhibit? Select all that apply. A. Change in quality of voice B. White or red patch in the mouth C. Purulent Sputum D. Change in the fit of dentures

Correct answer/s: ___A, B,D____ Rationale: for each answer options-with Chapter & page number a) Rationale: Some patients initial symptoms associated with oral cavity cancer experience what feels like a lump in the throat or a change in the quality of voice. Chapter 26 & pg. 491. b) Rationale: Cancer of the oral cavity may initially show as a symptom of white or red patches in the mouth. Chapter 26 & pg. 491. c) Rationale: A common symptom of bacterial pneumonia is purulent sputum. Chapter 26 & pg. 481. d) Rationale: Cancer of the oral cavity may initially show as a symptom of change in the fit of dentures. Chapter 26 & pg. 491.

A patient is receiving discharge instructions following surgical excision of melanoma. What should the nurse include in the teaching plan? Select all that apply. a) "You must use a sunscreen of at least 50 SPF when you will be exposed to sunlight." b) "Use waterproof sunscreen if you will be swimming outdoors, and it does not need to be reapplied immediately because it is water resistant." c) "Wear a hat, sunglasses, and a darker-colored, long-sleeved shirt when going outside." d) Avoid being out in the sun between 10:00 AM and 2:00 PM to reduce your risk of developing melanoma."

Correct answer/s: ___C, D__ Rationale: for each answer options-with Chapter & page number a) The general recommendation is that everyone should use daily sunscreen with a minimum of SPF 15. The patient can wear SPF 50, but it is not required that it is the minimum SPF they need to wear. Chapter 23 pg. 408. b) Sunscreen should be applied 20 to 30 minutes before going outdoors, even in cloudy weather. Sunscreens are not "waterproof" and must be reapplied immediately after swimming. Chapter 23 pg. 408. c) To avoid damaging effects of sun wear sunglasses, a large-brimmed hat, and a darker-colored, long-sleeved shirt of a tightly woven fabric or carry an umbrella. Chapter 23 pg. 408. d) The greatest risk for sun damage is between the hours of 10:00 AM and 2:00 PM. This is when 80% of UV rays occur. Chapter 23 pg. 408.

1. A 21 year old patient is being prepared for an appendectomy, what effects of general anesthesia should you inform the patient to expect? -Select all that applies a) Loss of sensation with loss of consciousness b) Skeletal muscle relaxation c) Sedative, anxiolytic, and/or analgesic drugs used d) Topical applications may be aerosolized or nebulized

Correct answer/s: __a, b_____ Rationale: for each answer options-with Chapter & page number a) Occurs under general anesthesia Chpt 18 pg. 323 b) Occurs under general anesthesia Chpt 18 pg. 323 c) Occurs with moderate, deep, and monitored anesthesia, not general anesthesia Chpt 18 pg. 323 d) Used under local anesthesia, not general anesthesia Chpt 18 pg. 323

The nurse meets with a 65-year old client prior to coronary artery bypass graft surgery. The nurse knows that the primary purposes of the preoperative interview are to: (select all that apply) a) Assess the patient's emotional state, readiness for surgery, and expectations about the surgery. b) Perform a mini-cog screening as a cognitive baseline to use postoperatively. c) Provide and clarify information about the planned surgery. d) Obtain information about the patient's health, including drug and food allergies.

Correct answer/s: __a, c, d____ Rationales: a) CORRECT - The preoperative interview can occur in advance or on the day of surgery. One of the primary purposes of the patient interview is to assess the patient's emotional state and readiness for surgery, including his or her expectations about the surgical outcomes. - Chapter 17, page 302. b) Performing a mini-cog screening is not one of the primary purposes of the preoperative interview. The mini-cog is used as a brief assessment tool for cognitive impairment. It can be quickly administered and can guide the need for further evaluation. - Chapter 17, page 302 Chapter 59, page 1408 (Table 59-9). c) CORRECT - One of the primary purposes of the patient interview is to provide and clarify information about the planned surgery. - Chapter 17, page 302. d) CORRECT - One of the primary purposes of the patient interview is to obtain the patient's health information, including drug and food allergies. - Chapter 17, page 302.

Early symptoms of Paget Disease include: Select all that apply. a. Markedly elevated serum alkaline phosphatase b. Gradually developing bone pain c. Fatigue d. Waddling gait e. Increased height

Correct answer/s: _b, c, d______ Rationale: for each answer options-with Chapter & page number a) Markedly elevated serum alkaline phosphatase is common in advanced disease, not an early symptom. (Ch. 63 pg. 1514) b) Gradually developing bone pain is an early symptom of Paget Disease. (Ch. 63 pg. 1514) c) Fatigue is an early symptom of Paget Disease. (Ch. 63 pg. 1514) d) Waddling gait is an early symptom of Paget Disease. (Ch. 63 pg. 1514) e) Patient's complain that they are becoming shorter, not getting taller. (Ch. 63 pg. 1514)

A nurse is assessing patients at the start of her shift. She determines which patients are at risk for candidiasis: (Select all that apply) a) 52-year-old obese female with increased sweat production b) 30-year-old male with HIV c) 28-year-old female with a fractured humerus d) 82-year-old female receiving chemotherapy for lung cancer

Correct answer/s: a, b, d Rationale: a) Candidiasis commonly appears in warm, moist areas of the body. Increased sweat production may contribute to the optimal environment for yeast growth (chapter 23, page 418). b) Immunosuppression (e.g., from HIV infection, chemotherapy, radiation, and organ transplantation) allows yeast to grow uninhibited and become pathogenic (chapter 23, page 418). c) Candidiasis commonly appears in warm, moist areas of the body. Immunosuppression (e.g., from HIV infection, chemotherapy, radiation, and organ transplantation) allows yeast to become pathogenic (chapter 23, page 418). Broken bones do not normally increase the risk of a candidiasis infection. d) Immunosuppression (e.g., from HIV infection, chemotherapy, radiation, and organ transplantation) allows yeast to become pathogenic (chapter 23, page 418).

1. The nurse in an urgent care clinic assess a patient who complains of the abrupt onset of the following symptoms: fever of 102.4 degrees F orally, headache, severe body aches and pains, fatigue, weakness, exhaustion, chest discomfort, and sore throat. The patient's symptoms began approximately twelve hours ago. There have been regional reports of Influenza A infections for the past two weeks. The nurse knows the following actions are appropriate (Select all that applies.). a) The nurse will anticipate that the patient has Influenza A based on the patient's symptoms and will be prepared to collect a culture via a throat swab. b) The nurse knows a rapid flu test or viral culture can be used since the patient is within the first 48 hours of the onset of symptoms. c) The nurse will encourage the patient to continue their daily routine like going to work and socializing because the patient is no longer contagious. d) The nurse will prepare to educate the client regarding symptom relief through rest, hydration, analgesics, and antipyretics, and shortening the length of the flu by treatment with an antiviral drug.

Correct answer/s: a, b, d Rationale: for each answer options-with Chapter & page number a) Rationale: Chapter 26, page number 481: Influenza is typically diagnosed based on the patients report of symptoms, clinical assessment, and knowledge of other cases of influenza in the community. Throat cultures may be collected but can take up to 3-10 days for results. The viral culture can identity which strain of the virus is present and the data will be used in the formulation of the flu vaccine for the following season. b) Rationale: Chapter 26, page number 481: The rapid flu test is best used within the first 48 hours of the onset of symptoms. The procedure takes place at the clinic and results are available within 30 minutes or sent to a laboratory and available the same day. Some results are occasionally false-positive. c) Rationale: Chapter 26, page number 481: The influenza virus has an incubation period of 1 to 4 days with peak transmission risk starting approximately 1 day before the onset of symptoms and continuing for 5 to 7 days after a person first becomes sick. This patient is currently in peak transmission and will be contagious for the next several days. The patient should stay home, keep their hands washed and avoid close contact with healthy people. d) Rationale: Chapter 26, page number 481: Rest and hydration will help with general fatigue, weakness, and exhaustion. Use acetaminophen or ibuprofen for headaches, aches, pain, and fever. Antiviral treatment can be given within 24-48 hours of onset and will help shorten the length of illness.

A patient diagnosed with tuberculosis is initially prescribed the 4-drug regimen of isoniazid, rifampin, pyrazinamide, and ethambutol. The nurse should educate the patient to contact the HCP if the following should occur-Select all that applies a) Yellowing of the skin and eyes b) Ringing in the ears c) Decreased visual acuity d) Gingival bleeding

Correct answer/s: a, c, and d a) Rationale: Hepatitis is an adverse effect of isoniazid - Ch. 27, Pg. 509 b) Rationale: Ototoxicity is an adverse effect of the tuberculosis drug streptomycin but not isoniazid, rifampin, pyrazinamide, or ethambutol - Ch. 27 & Pg. 509 c) Rationale: Ocular toxicity is an adverse effect of ethambutol - Ch. 27, Pg. 509 d) Rationale: Thrombocytopenia is an adverse effect of rifampin - Ch. 27 & Pg. 509

A patient complains progressive increase in lower back pain, the nurse recognizes several potential causes, including: a) Daylight savings b) Recent weight gain causing obesity c) Regular yoga routines d) Recent cold and wet weather

Correct answer/s: b, d, a) Potential causes for lower back pain are listed in chapter 11 on page 1504, daylight savings is not a listed cause. b) Obesity causes chronic strain on the patients back, which can lead to an increase in chronic pain. Chapter 11, page 1504. c) Yoga can actually reduce lower back pain if done properly. Chapter 11, page 1504 d) Cold and wet weather has been shown to aggravate chronic lower back pain. Chapter 11, page 1504.

Possible complications of cystic fibrosis include? Select all that applies a) Cor pulmonale b) Bronchiectasis c) Diarrhea d) Diabetes mellitus

Correct answer: A, B, & D

The nurse is developing a plan of care for a patient who is 62 years old and has just suffered a right-brain stroke. The patient is incontinent of urine and tells you during your assessment that he usually skips breakfast because he doesn't have much of an appetite until lunchtime. In order to prevent the patient from forming a pressure ulcer, which nursing interventions will you add to your plan of care? (Select all that apply) a.Reposition the patient in a "side-back-side" pattern with a maximum duration of 2 hours per position, but never position the patient on the weak or paralyzed side b. Float the patient's heels with pillows c. Assist patient to commode or bedpan at regular intervals d. Educate the patient and the caregiver about the importance of consuming a diet adequate in calories, protein, fluids, vitamins and minerals e. Discourage early mobility and let patient know that rest should be their top priority right now

Correct answer: A, B, C, D a. An example of a position change schedule to prevent skin breakdown is "side-back-side", and the patient should be positioned on their weak or paralyzed side for no more than 30 minutes - Ch 57, pg 1360 b) Pillows can be used under lower extremities to reduce pressure on the heels - Ch 57, pg 1360 c) Care for the incontinent patient should include assisting them to the commode or bedpan at regular intervals. Ch 45- pg 1059 d) Oral feedings must be adequate in calories, protein, fluids, vitamins and minerals to meet the patient's nutritional requirements and to enhance ulcer healing - Ch 11, pg 175 e) Early mobility should be encouraged to prevent skin breakdown - Ch 57, pg 1360

1. A patient being discharged from the rehab center receives education on epistaxis care. Which statement by the patient lets the nurse know the teaching was effective? (select all that apply) A. I am going to focus on applying pressure directly to the soft lower portion of my nostrils when bleeding occurs. B. When the epistaxis starts, I am going to sit down and lean my head back. C. I hope my nosebleeds do not become too severe because nasal packing is painful. D. I should sneeze with my mouth open now that I have had the packing removed.

Correct answer: A, C, D A. With epistaxis, it is important to apply the pressure to the lower portion of the nose (nostrils) for 5-15 minutes before seeking medical assistance. Ch. 26 P. 477 B. Rather than leaning the head back, the patient is supposed to sit down and lean the head forward to decrease the amount of blood going down the thought Ch. 26 P. 746 C. Nasal packing is painful because sufficient pressure needs to be applied to stop the bleeding. Patients may receive an opioid analgesic for pain. Ch. 26 P. 477 Patients who are prone to nosebleeds should receive instruction with their caregiver on the use of saline nasal spray and/or humidifier, sneezing with the mouth open, and avoiding aspirin or NSAIDs. Sneezing with the mouth open decreases the pressure forced through the nose and is good practice after a patient has had nasal packing. Ch. 26 P.746

A patient diagnosed with Acute Respiratory Distress Syndrome has a CBC lab completed with white blood cells at 13 thousand/mcL. Which phase of the disease is this patient likely in? A. Injury Phase B. Reparative/Proliferative Phase C. Exudative Phase D. Fibrotic Phase

Correct answer: B Pg. 1622-1623

An older patient who has been using corticosteroids long-term complains to the nurse about out a decrease in vision, abnormal color perception, and glare. Which condition does the nurse suspect? a) Refractive error b) Cataract c) Age-related macular degeneration d) Primary open-angle glaucoma

Correct answer: B Rationale: for each answer options-with Chapter & page number a) Rationale: (Ch. 21 p. 367) Major symptom is blurred vision, ocular discomfort, eyestrain or headaches b) Rationale: (Ch. 21 p. 374) Patient with cataracts may complain of a decrease in vision, abnormal color perception, and glare. Glare is due to light scatter caused by the lends opacities and may be significantly worse at night when pupil dilates. Most cataracts are age- related (senile cataracts). Some drugs such as corticosteroids are an associated factor to cataracts. c) Rationale: (Ch. 21 p. 378) Patient may complain of blurred and darkened vision, scotomas (blind spots), and metamorphopsia (distortion of vision) d) Rationale: (Ch. 21 p. 379) Visual clinical manifestations around eye include seeing colored halos around lights, blurred vision, and ocular redness.

Which of the following individuals is most likely to be diagnosed with a malignant bone tumor? a) A 50 year old with a history of hypertension b) A 25 year old with previously treated for cancer using radiation treatments c) A 60 year old with Paget's disease d) A 10 year old who is immunocompromised

Correct answer: B Rationale: a) Rationale: Chapter 63, page 1500-1501 Sarcomas occur most often during childhood and young adulthood. b) Rationale: Chapter 63, page 1500-1501 This individual is within the age range of "child to young adult". Another risk factor for this person is that they have had prior radiation treatment which is often associated with forming a malignant bone tumor. c) Rationale: Chapter 63, page 1500-1501 Paget's disease is associated with the formation of a sarcoma, but they are outside of the age range that is normally affected. d) Rationale: Chapter 63, page 1500-1501 This person is within the age range, but being immunocompromised does not cause malignant bone tumors.

A patient presents to the Emergency Department with increased blood pressure, tachypnea, hypoxic, and increased heart rate. When conducting your nursing assessment, you note tracheal deviation and absent lung sounds on the left side. As the nurse, you suspect the patient has: a. Pleural Effusion b. Tension Pneumothorax c. Cardiac Tamponade d. Flail Chest

Correct answer: B. Tension Pneumothorax Rational: a) Pleural Effusion occurs buildup of fluid in the pleural space. Pleural Effusions will not have tracheal deviation (Chapter 25 pg. 456) b) Tension Pneumothorax occurs when air builds up in the pleural space causing an increased pressure. The increased pressure will cause the trachea to move to the unaffected side of the lungs. When the trachea is not aligned this is called tracheal deviation. The lung sounds on the affected side with the Tension Pneumothorax will be diminished or absent (Chapter 27, page 520). c) Cardiac Tamponade occurs when blood rapidly collects in the pericardial sac and compresses the myocardium. Cardiac Tamponade manifests with muffled and distant heart sounds, low blood pressure instead of high blood pressure and will not have tracheal deviation or absent lung sounds on the affected side (Chapter 27, page 520). d)Flail Chest occurs with a fracture to two or more adjacent ribs in two or more places with the loss of chest wall stability. Paradoxical movement of the chest wall is a main manifestation of Flail chest, not tracheal deviation (Chapter 27, page 520).

The nurse is teaching a 20-year-old patient with cystic fibrosis the importance of adequate nutrition. The nurse recognizes that further teaching is required if the patient says which of the following? a) "I will need to take Pancreaze before each meal and snack I have." b) "If I am sweating a lot I will add dietary salt to my diet." c) "A balanced diet of low fat & high protein is best for my health." d) "I should use caloric supplements when my diet is inadequate."

Correct answer: C

After a lesson on fibromyalgia, a nursing student is summarizing fibromyalgia pain. The instructor knows the student needs clarification when the student states: a) "Fibromyalgia pain is widespread and may be described as burning, tender, or stiff." b) "Pain from fibromyalgia is chronic and may lead to decreased quality sleep and activity intolerance." c) "Patients with Fibromyalgia may feel most stiff in the mornings and often experience less pain as the day progresses." d) "Like other chronic pain syndromes, patients with Fibromyalgia may report cognitive dysfunction like memory lapses or mood disorders such as anxiety and depression."

Correct answer: C Rationale: a) This is a true statement. Patients may feel stiff, particularly at tender points and as pain progresses it may become burning in sensation. Pain differs for each patient and may fluctuate during a single day (Chapter 64, p. 1547). b) This is a true statement. For a diagnosis of Fibromyalgia, pain must be present for at least 3 months, indicating that pain is chronic, and must not be the result of another disease process. Sleep disturbances are common in patients with fibromyalgia, either due to pain and discomfort, hypothalamic-pituitary-axis dysfunction, inadequate sleep-dependent hormone levels, or abnormal neurotransmitter levels (Chapter 7, p. 92 Chapter 64, p. 1547). c) This statement is incorrect, indicating that the student would benefit from clarification. While stiffness may peak in the morning, particularly after nonrestorative sleep, pain may worsen or improve through the course of a day (Chapter 64, p. 1547). This is a true statement. Anxiety, depression, insomnia, non-restorative sleep, Restless Leg Syndrome, difficulty concentrating, memory lapses, irritability, allodynia, and sensitivity to painful stimuli are common manifestations in patients with Fibromyalgia (Chapter 64, p. 1547).

A nurse is working with a patient recently diagnosed with Huntington's disease. Prior to discharge, the nurse is evaluating if education about the disease has been successful. The nurse would know that further education about Huntington's disease is necessary if the patient makes which of the following statements? A. "I should consider having my children tested for this disease." B. "I don't have any advance directives. I should talk with my partner to decide what needs to be done as my disease progresses." C. "I know someone suffering from Parkinson's disease who takes Sinemet to help with their tremors. I should talk with my physician to be prescribed something similar for my motor issues." D. "My physician prescribed me Tetrabenazine. I should probably talk to them about my history of depression before I begin this medication."

Correct answer: C a) Rationale: This is an appropriate response. Huntington's disease is a genetically transmitted and is autosomal dominant. The offspring of an individual with Huntington's disease is at a 50% risk of inheriting this disease. Furthermore, Huntington's disease is often diagnosed in individuals after they have had children, so genetic testing for the patient's children should be encouraged. Chapter 58, page 1395. b) Rationale: This is an appropriate response. Huntington's disease has no cure, and death can be expected around 10 to 20 years after a diagnosis is made. End-of-life issues are important to consider in patients with Huntington's disease. Advanced directives should be initiated. As the disease progresses, further discussion is necessary as disability increases. Chapter 58, page 1396. c) Rationale: This is incorrect. While both Huntington's disease and Parkinson's disease result from a pathology within the basal ganglia and extrapyramidal motor systems, the pathophysiology of Huntington's disease is related to a deficiency of acetylcholine (ACh) and ỿ-aminobutyric acid (GABA). Chapter 58, page 1395. Carbidopa-levodopa (Sinemet) is indicated in Parkinson's disease to replace dopamine in the basal ganglia related to a dopamine deficiency. Chapter 58, page 1390. Dopamine replacement in Huntington's disease is contraindicated and would exacerbate the disease. d) Rationale: This is an appropriate response. Tetrabenazine is often a first line treatment for patients with Huntington's disease. The mechanism of action is to inhibit the excess of dopamine that is characteristic of the disease to assist with choreic movements. However, if chorea is accompanied with other symptoms, such as depression, alternative treatment modalities may be considered so that a lack of dopamine does not exacerbate depression. This is particularly important to consider because the psychiatric deterioration that can accompany Huntington's disease is responsible for suicide being the second leading cause of death in individuals with Huntington's disease. Chapter 58, page 1396.

The nurse knows that the primary focus of glaucoma therapy is to: A. Recover lost vision B. Reduce intraocular inflammation C. Reduce opacity within the lens. D. Prevent optic nerve damage

Correct answer: D Rationales: a) Incorrect - Continued drug treatment controls but does not cure glaucoma there is no cure. (Ch 21 pg 380) b) Incorrect: Glaucoma is characterized by increased intraocular pressure caused by a greater inflow of aqueous humor and outflow, intraocular inflammation is not a symptom of glaucoma. (Ch 21 pg 379) c) Incorrect: Opacity within the lens is a cataract. (Ch 21 pg 373) d) Correct: The primary focus of glaucoma therapy is to keep the IOP low enough to prevent the patient from developing optic nerve damage. (Ch 21 pg 379-380)

The nurse of a small town emergency department is caring for a patient who was admitted with severe chest pain. An ECG was done and indicates an ST elevated myocardial infarction has occurred. Which of the following is the best action by the nurse to do next? a) Remove the patients IV line before they get their MRI. b) Administer 162 mg of aspirin within 90 minutes of arrival to Emergency department. c) Administer tPA and monitor patient for bleeding over the next 48 hours. d) Immediately begin thrombolytic therapy and prepare the patient to be transferred.

Correct answer: D Rationale: for each answer options-with Chapter & page number a) An MRI is not indicated for this patient an instead multiple IV lines should be started in preparation for thrombolytic therapy (Chap 33, pg 724). b) Aspirin should be given with NTG, however, the nurse should confirm that the patient hasn't already taken a dose of aspirin or NTG before arriving at the emergency department (Chap 33, pg 722). c) tPA is a fibrinolytic drug used for the treatment of ischemic stroke (Chap 57, pg 1355). d) For any individual with a confirmed STEMI the best treatment is to do a PCI. However, if a facility is not equipped to perform a PCI then a patient should be placed on thrombolytic therapy within 30 minutes upon arrival to help open up the occluded artery and limit infarction size until they can be transferred to a critical care unit (Chap 30, pg 722).

The home health nurse is providing care to a 32 year old man who has suffered a spinal cord injury and has paralysis of his lower extremities. The nurse discovers a stage III pressure ulcer on his coccyx. The patient's wife is his primary caregiver. The nurse knows that additional teaching is necessary when the patient says: a)"I will make sure to use my trapeze bar when I'm in bed to reposition myself by lifting rather than sliding" b) "It is very important that my wound never dries out but stays somewhat moist" c) "I will make sure to eat meals that are high in calories, protein, vitamins and minerals to help my wound heal" d) "Cleaning the ulcer every day with hydrogen peroxide is very important"

Correct answer: D a) Lifting to reposition rather than sliding is indicated because sliding down in bed can cause shear - Ch 11, page 175 b) A pressure ulcer should be kept slightly moist, rather than dry, to enhance re-epithelialization - Ch 11, page 175 c) Oral feedings must be adequate in calories, protein, fluids, vitamins and minerals to meet the patient's nutritional requirements - Ch 11, pg 175 d) Hydrogen peroxide is a cytotoxic agent and may kill or damage cells, especially cells that are necessary for wound healing like fibroblasts. Therefore, using hydrogen peroxide is contraindicated - Ch 11, page 175

Which of the following is not an appropriate and common medication given for a patient with a pulmonary embolus? A. Enoxaparin (Lovenox) B. Alteplase (Activase) C. Warfarin (Coumadin) D. Lurasidone (Latuda)

Correct answer: D. Lurasidone (Latuda) a) Rationale: Enoxaparin is an anticoagulant which is used as an immediate drug therapy when a pulmonary embolus is suspected because of its long duration therapeutic effect to inhibit coagulation factors (Lewis et al., 2017, p. 530). b) Rationale: Alteplase is a fibrinolytic agent which is commonly used to help dissolve the pulmonary embolus and the source of thrombus, therefore decreasing the likelihood of recurrent embolus (Lewis et al., 2017, p. 530). c) Rationale: Warfarin is another type of anticoagulant that reduces the function of Vitamin K in the clotting cascade, therefore inhibiting coagulation (Lewis et al., 2017, p. 530). d) Rationale: Lurasidone is an atypical antipsychotic which is not indicated for use in a patient with a pulmonary embolus (Lewis et al., 2017, p. 530).

A patient has been diagnosed with neural presbycusis. This is most likely characterized by: A. Loss of high-pitched sounds. Little effect on speech understanding. B. Loss of speech discrimination. C. Uniform loss for all frequencies accompanied by recruitment D. Hearing loss increases from low to high frequencies. Speech discrimination affected with higher-frequency losses.

Correct answer: __B_____ Rationale: for each answer options-with Chapter & page number a) This describes the characteristics of sensory presbycusis (Ch. 21, p 391, Table 21-15) Sensory presbycusis is caused by atrophy of the auditory nerve and loss of sensory hair cells. These patients have a good response to sound amplification. b) Neural presbycusis is characterized by a loss of speech discrimination. Amplification alone is not sufficient to treat this. It is caused by degenerative changes in cochlea and spinal ganglion. c) This describes the characteristics of metabolic presbycusis (Ch. 21, p 391, Table 21-15) Metabolic presbycusis is caused by atrophy of blood vessels in wall of cochlea with interruption of essential nutrient supply. These patients usually have a good response to hearing aids d) This describes the characteristics of cochlear presbycusis (Ch. 21, p 391, Table 21-15) Cochlear presbycusis is caused by stiffening of basilar membrane, which interferes with sound transmission in the cochlea. It can be helped by appropriate forms of amplification

A patient presents to the clinic with an erythematous, pearly, sharply defined papule with semitranslucent borders on their left arm. The nurse would expect which diagnosis from the physician? a) Psoriasis b) Actinic Keratosis c) Erysipelas d) Basal Cell Carcinoma

Correct answer: __D.___ a) Psoriasis is a sharply demarcated, silvery, scaling plaques on reddish colored skin commonly on the scalp, elbows, knees, palms, soles, and fingernails. Chapter 23 pg. 419 b) Actinic Keratosis is a flat or elevated, dry, hyperkeratotic scaly papule. Chapter 23 pg. 413. c) Erysipelas is a red, hot, sharply demarked plaque that is indurated and painful. Chapter 23 pg. 414. d) Basal Cell Carcinoma is a small, slowly enlarging papule with semitranslucent or "pearly" borders. In superficial Basal Cell Carcinoma, the papule is erythematous, pearly, sharply defined, and barely elevated. Chapter 23 pg. 412.

A patient has presented to the Emergency Department with acute airway obstruction related to severe facial trauma and severe upper airway swelling from a motor vehicle accident and will require prolonged airway management. Which of the following interventions would you anticipate the physician to carry out to reestablish a patent airway for long term use? A. Cricothyroidotomy B. Endotracheal Intubation C. Heimlich maneuver D. Tracheostomy

Correct answer: __D_____ A. While this is a useful procedure, it is not typically used in long term airway management B. Swelling, facial trauma, and upper airway obstruction can prevent ET intubation thus requiring emergent surgical tracheostomy C. Only indicated if airway obstruction is from food or other objects- D. When swelling, trauma, or upper airway obstruction prevents ET intubation, an emergent surgical tracheostomy is performed

What is the first action the nurse should take when caring for a patient with a tension pneumothorax? a) Ask the patient what they were doing right before the pneumothorax occurred to determine what caused the trauma. b) Elevate head of bed to 30 degrees to make breathing easier. c) Provide proper analgesia to manage pain. d) Perform needle decompression followed by chest tube insertion with chest drainage system.

Correct answer: __D_____ Rationale: for each answer options-with Chapter & page number a) Tension pneumothorax is a medical emergency. If the tension in the pleural space is not relieved the patient is likely to die. First thing that must be done is to relieve this increased pressure. Ch 27 pg 520 b) Tension pneumothorax is a medical emergency. If the tension in the pleural space is not relieved the patient is likely to die. The first thing that must be done is to relieve this increased pressure. Ch 27 pg 520 c) Tension pneumothorax is a medical emergency. If the tension in the pleural space is not relieved the patient is likely to die. First thing that must be done is to relieve this increased pressure. Ch 27 pg 520 d) Tension pneumothorax is a medical emergency. If the tension in the pleural space is not relieved the patient is likely to die. First thing that must be done is to relieve this increased pressure. Ch 27 pg 520 (Table 27-20)

Which of the following patient would the nurse consider highest risk for the development of Coronary Artery Disease? a) A 78 year old male with a history of uncontrolled diabetes and is currently taking simvastatin. b) A 45 year old female with a history of Atrial fibrillation. c) A 25 year old male with a temp of 100 and a history of elevated cholesterol. d) A 57 year old female with an elevated BG in from a motor vehicle accident.

Correct answer: ___A___ Rationale: for each answer options-with Chapter & page numbe a) This patient is considered the highest risk for CAD due to his age, gender, and health history. Found on page 703 in chapter 33 b) This patient is not considered the highest risk for Coronary Artery Disease specifically. Although Atrial fibrillation should be evaluated, it is not considered the most urgent. Found on page 766 in chapter 35 c) This patient is not considered the highest risk due to age and the slightly elevated temperature. The goal of this question is to allow the student to recognize the risk factors specifically associated with Coronary Artery Disease. This patient does have one risk factor but is not considered the most urgent. Found on page 704 Chapter 33 d) Although this patient has an elevated Blood Glucose, the point of this answer option is to allow the student to recognize the elevated Blood Glucose most likely has nothing to do with her heart rather the stressful experience the patient has just encountered. It is important when screening for possible Coronary Artery Disease cases that the nurse be able to recognize what qualifies as a risk factor and what does not.

A nurse is providing patient and caregiver teaching about the management of sinusitis. The charge nurse would need to intervene if a nurse makes which statement? a) Stay well hydrated and drink six to eight glasses of water a day. b) Report a temperature of 100.4 degrees or higher. c) It is best to sleep on your side to help your sinuses drain. d) Take hot showers multiple times a day.

Correct answer: ___C____ Rationale: for each answer options-with Chapter & page number a) Staying well hydrated will help loosen secretions. - Chapter 26 Pg. 483 b) A temp of 100.4 degrees or higher can indicate an infection and should be reported to their physician. - Chapter 26 Pg. 483 c) It's important to for the client to sleep with their head elevated to help sinuses drain and help reduce congestion.: for each answer options- Chapter 26 Pg. 483 d) Hot showers will help promote sinus drainage. - Chapter 26 Pg. 483

Which risk factor makes it most likely that the patient will develop Head and Neck Cancer? A. Human Papillomavirus Infection B. Alcohol Consumption C. Tobacco Products (including smokeless tobacco) D. Sun Exposure

Correct answer: ___C____ Rationale: for each answer options-with Chapter & page number a) Rationale: Head & neck cancers in patients younger than fifty years are often associated with human papillomavirus (HPV) infection. Chapter 26 & pg 491. b) Rationale: Excessive Alcohol consumption is also a major risk factor. Chapter 26 & pg 491. c) Rationale: Eighty- five percent of head and neck cancers are caused by tobacco (including smokeless tobacco) Chapter 26 & pg 491. d) Rationale: Other risk factors include sun exposure, radiation therapy to the head and neck. Chapter 26 & pg 491

In the Post-Anesthesia Care Unit (PACU), what task can the registered nurse delegate to a Licensed Practical Nurse (LPN)? a) Evaluating for the return to consciousness and ability to maintain airway and breathing b) Providing discharge teaching for patient and caregiver c) Administer and titrate O2 based on agency protocols d) Provide hand-off report about patient status when transferring patient to RN on clinical unit

Correct answer: ____C___ Rationale: a) Evaluating airway, breathing, and level of consciousness is part of assessment to see if there are any postoperative complications such as atelectasis, hemodynamic instability, cognitive dysfunction, pain, fluid and electrolyte imbalance, and fever/hypothermia among others. Assessments are responsibilities of the RN (p334). b) RN is responsible for developing an individualized care plan, implementing the plan, and discharge teaching to the patient and caregiver (p334). c) In the PACU, the RN can delegate tasks such as administering or titrating O2 as well as giving analgesics and IV fluids per agent policy (p334). d) The RN develops and implements an individualized plan of care based on the patient's background, risk factors, and potential consequences. Therefore, the RN will have the information necessary to satisfactorily report when transferring the patient (p334).

The nurse provides discharge instructions to the 60-yr-old male patient with Paget disease. Which statement by the patient indicates a need for further teaching? a) I will limit my activity to decrease risk for further bone loss. b) I may need to wear a brace to relieve back pain. c) I can take Naproxen as needed for pain relief. d) I will call my son to help me lift boxes in my garage.

Correct answer: __a_____ Rationale: for each answer options-with Chapter & page number a) Keep the patient as active as possible to slow demineralization of bone from disuse or extended immobilization. (Ch. 63 pg. 1514) b) The Patient may need to wear a corset or light brace to relieve back pain and provide support when upright. (Ch. 63 pg. 1514) c) Pain is usually managed by NSAIDs. (Ch. 63 pg. 1514) d) Discourage lifting and twisting. (Ch. 63 pg. 1514)

The nurse is performing a preoperative assessment of a 22-year-old college athlete prior to surgery to repair a torn anterior cruciate ligament (ACL). When the client expresses anxiety due to the uncertainty of the upcoming surgery, which response by the nurse is best: a) "Don't worry, your surgeon has performed this surgery thousands of times. I am sure you will be fine." b) "Preparing for surgery can be stressful, and it is normal to feel anxious. What seems to be concerning you?" c) "You don't need to worry, you are only having outpatient surgery for a torn ACL." d) "Surgery is a stressful event, but you will receive anesthesia and analgesia during surgery so there is no need to worry."

Correct answer: __b____ Rationales: a) Most people are anxious when facing surgery because of the unknown. You can decrease some anxiety for the patient by providing information about what to expect. The nurse should not provide false reassurance to the patient. Chapter 17, Pages 302-303. b) CORRECT - This response is therapeutic since it elicits an open response from the patient. Part of the nurse's role in assessing a preoperative patient involves psychologically preparing the patient for surgery by assessing the patient potential or actual stressors that could negatively affect surgery (Table 17-2). Communicate all concerns to the appropriate surgical team member, especially if the concern requires intervention later in the surgical experience. - Chapter 17, Page 302-303. c) Anxiety can arise from lack of knowledge. This can range from not knowing what to expect to uncertainty about the outcome and may result from past experiences or stories heard through friends or the media. Inform the surgeon if the patient requires any additional information or if anxiety is excessive. - Chapter 17, Page 302-303. d) The nurse should ask the patient about their cause for anxiety. Also, the use of common language and avoidance of medical jargon are essential. Use words and language that are familiar to the patient to help the patient understand the surgery. Familiar language also helps reduce preoperative anxiety. - Chapter 17, pages 302-303.

A patient is diagnosed with Tinea Unguium (Onychomycosis). The nurse recognizes that the most effective treatment for this infection is: a) The use of black fingernail polish to deprive the fungus of light b) Systemic treatment with oral antifungals c) Localized treatment with Topical antifungals d) Increased skin hygiene to keep area clean and dry

Correct answer: b Rationale: a) Covering the infected nails with black fingernail polish is not a proven treatment for Tinea Unguium (Onychomycosis) (chapter 23, page 418). b) Oral antifungals such as terbinafine (Lamisil) and itraconazole (Sporanox) produce systemic effects and are the first-line of defense against Tinea Unguium (Onychomycosis) (chapter 23, page 418). c) Topical antifungal cream or solution has minimal effectiveness and is used only if unable to tolerate systemic treatment (chapter 23, page 418). d) Skin hygiene is important but not the most effective treatment against Tinea Unguium (Onychomycosis) (chapter 23, page 418).

A 54-year old Asian male was brought to the ER by his spouse and has been hospitalized for tuberculosis and is receiving drug therapy. He asks about when he can be discharged. What is the best response from the nurse? a) You can be discharged when negative culture results are received from your lab work b) You can be discharged home with positive cultures when you respond clinically to treatment and do not pose a risk to others c) You will need to stay at the hospital for a minimum of one month d) It just depends, every discharge outcome is different for each tuberculosis patient

Correct answer: b a) Rationale: Negative cultures are needed to declare the patient not infectious, but not necessary to be discharged home - Ch. 27, Pg. 511 b) Rationale: If the patient is responding clinically, the patient's contacts (in this case the spouse) have already been exposed, and the patient is not posing a risk to others, the patient can be discharged home with positive cultures - Ch. 27, Pg. 511 c) Rationale: There is no set minimum amount of time the tuberculosis patient must be hospitalized - Ch. 27, Pg. 511 d) Rationale: While this is a correct response, it is vague and ineffective and is not the appropriate best response - with Ch. 27, Pg. 511

A 70-yr old male patient arrives at the clinic with symptoms of fatigue, anorexia, weight loss, and peripheral edema related to his recent diagnosis of chronic constrictive pericarditis without effusion. The nurse knows that she will likely need to prepare the patient for which procedure? a) Insertion of a cardiac pacemaker b) Pericardiectomy c) Pericardial biopsy d) Pericardiocentesis

Correct answer: b a) A cardiac pacemaker is an electronic device that is used when the normal conduction pathway of the heart is damaged. Chronic pericarditis often results in thickening of the pericardium with non-specific changes in electrocardiogram (ECG) findings. Although symptoms of chronic pericarditis mimic those of heart failure, the dysrhythmias seen with heart failure are not associated or caused by chronic pericarditis. Chapter 36, Page 787 Chapter 34, Page 743 b) The treatment of choice for chronic constrictive pericarditis is pericardiectomy, which is a complete resection of pericardium through median sternotomy with use of cardiopulmonary bypass. Improvement of symptoms can occur immediately or weeks after the procedure. Chapter 36, Page 787 c) A pericardial biopsy may be used to diagnose acute pericarditis, but not chronic pericarditis. Chapter 36, Page 784 - 787 d) A pericardiocentesis is usually done for pericardial effusion with acute cardiac tamponade, which the patient does not have. With pericardiocentesis, a needle is inserted into the pericardial space to remove fluid for relief of pressure on the heart and for fluid analysis. Chapter 36, Page 786

The nurse prepares to educate a community non-profit organization on the benefits of Influenza immunization. Which one of the following choices does the nurse include? a) It's reasonable for people to be reluctant to vaccinate because the flu vaccine can cause an influenza infection. b) Vaccination of healthy people decreases the incidence and risk of transmitting influenza to those who have less ability to cope with the effects of the flu. c) An alternative to immunization is supplementing with Zinc lozenges. d) Health care workers do not have to get immunized because they work in health care centers with low risk, healthy people.

Correct answer: b Rationale: for each answer options-with Chapter & page number a) Rationale: Chapter 26, page number 482: The flu vaccine is available as a Trivalent Inactivated Influenza vaccine or Live Attenuated Influenza vaccine. The flu vaccine does not cause an infection but does result in the production of antibodies against the virus contain in the vaccine. b) Rationale: Chapter 26, page number 482: Some people should not get the flu vaccine. It is contraindicated in people with a history of severe allergic reactions to previous flu vaccine and those with anaphylactic hypersensitivity to eggs. When healthy people vaccinate, it decreases the incidence of transmitting influenza to those who are vulnerable to infection and cannot be immunized. c) Rationale: Chapter 26, page number 482: Supplementing with Zinc lozenges does not produce antibodies that protect against the flu virus. Zinc is considered a complementary alternative therapy that when administered within 24 hours of the onset of symptoms can reduce the duration and severity of cold symptoms, but not the flu. d) Rationale: Chapter 26, page number 482: Health care workers are considered a high priority group for immunization because they can transmit influenza to high-risk people they provide care for within the health care system.

A nurse is educating a patient with a high risk of developing infective endocarditis. Which statement by the client reflects an accurate understanding of the condition? a) "It is important for me to avoid taking antibiotics because they put me at risk for developing infection with resistant organisms. " b) "I should be on the lookout for common manifestations of endocarditis including Roth's spots, sydenham's chorea, erythema marginatum, and subcutaneous nodules." c) "Though infective endocarditis is usually self-limiting and not dangerous to me, I would need to take antibiotics to prevent spreading it to others." d) "It could take weeks to eradicate the organism, and bed rest and valve replacement may become necessary." e) "For a definitive diagnosis, my cerebrospinal fluid would be collected 3 different times and saved for 3 weeks to look for slow-growing organisms."

Correct answer: d a) Rationale: Antibiotic treatment is typically needed for 4-6 weeks and may continue longer on an outpatient basis (Chapter 36, page 784). Additionally, prophylactic antibiotic use for at risk patients is common before certain procedures including dental work, surgery and others (Chapter 36, page 783). b) Rationale: While Roth spots are characteristic of endocarditis (Chapter 36, page 781), the other options are manifestations of rheumatic heart disease (Chapter 36, page 788). c) Rationale: Infective endocarditis is not-self limiting and requires antibiotics to prevent dangerous potential consequences including sepsis, heart failure, heart block and embolization to brain, lungs, limbs, kidneys, liver or spleen (Chapter 36, page 782). d) Rationale: Correct. Antibiotics may be prescribed for up to 6 weeks (Chapter 36, page 784). In certain situations, fungal infections for instance, a valve replacement may be needed in addition to drug therapy. While bed rest is not always prescribed, it may be necessary with prolonged fever or heart failure (Chapter 36, page 782). e) Rationale: Three blood cultures, not cerebrospinal fluid samples, may be collected in the diagnosis of infective endocarditis. Negative blood cultures may be retained to look for slow-growing organisms (Chapter 36, page 782).

A patient with suspected spinal cord injury arrives in the emergency department. Which of the following is the preferred diagnostic study to determine the location, degree of injury, and the degree of spinal canal compromise? a) Cervical x-rays b) MRI c) Comprehensive neurologic exam d) CT scan

Correct answer: d a) Rationale: Cervical x-rays are obtained when CT scan is not readily available. Visualizing C7 and T1 on cervical x-rays is often difficult, and the ability to fully evaluate cervical spine injury is compromised. Chapter 60, Page 1424 b) Rationale: MRI is used to assess soft tissue injury, neurologic changes, unexplained neurological deficits, or worsening neurologic condition. Chapter 60, Page 1424 c) Rationale: A comprehensive neurologic examination with assessment of the head, chest, and abdomen will help determine additional injuries or trauma. Chapter 60, Page 1424 d) Rationale: CT scan is the preferred imaging study to diagnose the location and degree of injury as well as the degree of spinal canal compromise. Chapter 60, Page 1424

The nurse knows the patient has not been properly educated regarding lower back pain if the patient makes which statement? a) I should place one foot on a low stool if I need to stand for long periods b) I should choose an office chair with a swivel base c) I need to sleep on my back with a 10-inch pillow under my knees d) I should carry weighty objects above the level of my elbows to decrease lower back strain

Correct answer: d a) When standing for a prolonged period, place one foot on a low stool to help maintain neutral pelvic position. Ch 11, Pg 1504. b) A chair with a swivel base allows its occupant to change leg position easily and frequently. Ch 11, Pg 1504. c) When sleeping on the back, patients with lower back pain should have a 10-inch pillow under their knees to flex both the knees and hips. Ch 11, Pg 1504. d) Patients with lower back pain should not lift anything above the level of their elbows because it will put more strain on their lower back. Ch 11, Pg 1504.

A patient comes to the hospital after being diagnosed with a malignant bone tumor. Which of the following are most important for the nurse to monitor? (select all that apply) a) Swelling b) Changes in circulation c) Fluctuation in blood pressure d) Decreased sensation

Correct answers: A, B, & D Rationale a) Rationale: Chapter 63, page 1501 A protrusion from the bone can cause swelling in the surrounding area. b) Rationale: Chapter 63, page 1501 Changes in circulation occur because of the swelling surrounding the bone tumor. Another factor that may cause decreased circulation arises when the patient is unwilling to participate in exercise or therapeutic activities because of weakness and fear or pain. c) Rationale: Chapter 63, page 1501 Fluctuations in blood pressure would be concerning to the nurse, but it is unrelated to the diagnosis of a bone tumor. d) Rationale: Chapter 63, page 1501 A decrease in sensation is a possible concern when a patient has a bone tumor because the tumor may be pressing against nerves surrounding the bone which may also result in severe pain.

A patient with fibromyalgia could benefit from nursing management that includes education or resources about (select all that apply) a) Stress management techniques b) Behavioral cognitive therapy and support groups c) Alternating cardiovascular exercise with weight or resistance training d) Appropriate sleep hygiene e) Long-acting opioids with prophylactic bowel care

Correct answers: A, B, D Rationale: a) Correct. Pain and associated symptoms, such as Irritable Bowel Disease, can cause significant stress. Since people living with chronic pain and patients with fibromyalgia may not cope well with stress which can lead to more tension or to more sleep disturbances which can cause further disruption to the hypothalamic-pituitary-adrenal axis and worsen symptoms (Chapter 6, p. 82 Chapter 7, p. 92 Chapter 64, p. 1547-1548). b) Correct. Support groups or individual therapy may benefit fibromyalgia patients who feel easily overwhelmed, isolated and alone, anxious, depressed, and stressed (Chapter 64, p. 1548). c) Incorrect. Patients with fibromyalgia may describe fatigue as limiting to their physical activity and may report a worsening of symptoms with exercise. Mild to moderate exercises or gentle stretching (yoga, tai chi, walking, etc.) are recommended to combat stiffness, depression, and muscle atrophy (Chapter 64, p. 1547-1548). d) Correct. Insomnia, Restless Leg Syndrome, Periodic Limb Movement Disorder, non-restorative sleep, and fatigue are some common sleep disturbances associated with fibromyalgia. Sleep hygiene can help to address behaviors that perpetuate insomnia. Excessive daytime sleepiness and non-restorative nighttime sleep may improve with the implementation of proper sleep hygiene practices including, but not limited to, setting a bedtime routine, going to bed when you are tired, and avoiding "screens" before bed (Chapter 7, p. 92- 93 Chapter 64, p. 1546-1548). e) Incorrect. Common drug therapy for chronic widespread pain associated with fibromyalgia includes, low-dose tricyclic antidepressants, selective serotonin reuptake inhibitors (SSRI), benzodiazepines, muscle relaxers, and/or over-the-counter analgesics including non-steroid anti-inflammatory drugs (NSAID). Side-effects of opioids such as sedation and constipation, which fibromyalgia patients may already be suffering from, make opioids a less ideal pharmacological option. In addition, long-term opioid use can cause opioid-induced hyperalgesia, a state of nociceptive sensitization, in which patients may experience increased pain. Since opioids tend to lose their effectiveness overtime, and fibromyalgia may be caused by a variety of hormonal or physiological neurologic dysfunctions, long-acting opioids are not the first line of treatment for patients with fibromyalgia (Chapter 8, p. 114-115, Chapter 64, p. 1546-1548).

A 70-yr-old male patient who arrives at the Emergency Department two days after being treated for a myocardial infarction is diagnosed with acute pericarditis. Which are typical assessment findings for acute pericarditis? Select all that apply a) Severe, sharp chest pain that is worse upon inspiration b) Heart murmur c) Grating, high-pitched sound heard upon auscultation of the heart d) Jugular venous distention (JVD)

Correct answers: a, c a) A typical sign and symptom of acute pericarditis is severe, sharp chest pain that is worse upon inspiration and when lying flat. The chest pain may also be referred to the trapezius muscle, shoulder, and upper back. Chest pain may be relieved by sitting up and leaning forward. Chapter 36, Page 785 b) A heart murmur is not typically associated with acute pericarditis. A heart murmur would be a separate diagnosis and may be the result of a heart valve disorder or abnormal blood flow patterns. Chapter 31, Page 668 c) The hallmark finding in acute pericarditis is a pericardial friction rub, which is a scratching, high-pitched, grating sound that is best heard using a stethoscope at the lower left sternal border of the chest (apex of the heart) with the patient leaning forward. A pericardial friction rub can be distinguished from pleural friction rub if the high-pitched, grating sound is heard when the patient holds his/her breath. Chapter 36, Page 785 d) Jugular venous distention (JVD) is not typically associated with acute pericarditis. However, JVD is one of the most prominent findings for patients who have chronic constrictive pericarditis. Chapter 36, Page 787

A student nurse is listening to a change-of-shift report when the clinical instructor asks her which of the patients may be at risk for developing infective endocarditis. The student correctly identifies which of the following patients at risk? (Select all that apply) a) A 27-year old woman who had a dilation and curettage procedure for the removal of retained tissue after a miscarriage at 13 weeks pregnant. b) A 46-year old man who fell from his roof while replacing shingles, sustaining an open, comminuted fracture of his right femur. c) A 91-year old male who presented with confusion and altered level of consciousness at admission and who has since developed a catheter associated urinary tract infection. d) A 20-year old male who attempted suicide after being diagnosed with Marfan syndrome. e) An 18-year old female who refuses to eat or participate in self-care since her admission for a heroin overdose.

Correct answers: a.b.c.d.e a) Rationale: Correct. While collecting subjective data, the registered nurse should question patients regarding recent gynecological procedures because they are a risk factor in the development of endocarditis (Chapter 36, page 783). b) Rationale: Correct. A patient with an open, comminuted femur fracture will require surgical intervention. Procedures involving musculoskeletal tissue place the individual at risk for developing infective endocarditis (Chapter 36, page 783). c) Rationale: Correct. Old age predisposes individuals to endocarditis, in part because of their increased likelihood of experiencing aortic stenosis. Additionally, hospital-acquired infections, such as a urinary tract infection, increase one's susceptibility to endocarditis (Chapter 36, page 781). d) Rationale: Correct. Marfan's syndrome is a known risk factor in the development of endocarditis (Chapter 36, page 781). e) Rationale: Correct. Intravenous drug use is a noncardiac circumstance that allows for potential development of endocarditis (Chapter 36, page 781).

A patient with a L1 spinal cord injury in undergoing rehabilitation. What should the nurse expect for the patient's ability? Select all that apply a) Decreased respiratory reserve b) Varying control of legs and pelvis c) Attendant care 10 hr/day d) Instability of lower back e) Full innervation of upper extremities

Correct answers: b, d, e a) Rationale: Decreased respiratory reserve would be expected in a spinal cord injury at or above the T1-6 level because of lack of innervation to diaphragm and intercostal muscles. A patient with a L1 spinal cord injury will have functional intercostal muscles resulting in increased respiratory reserve. Chapter 60, Pages 1421-1423, Table 60-2 b) Rationale: A patient with a L1 spinal cord injury will have varying control of legs and pelvis, depending on the degree and mechanism of injury. L1 spinal cord injuries may result in paraplegia with paralysis below the waist. Chapter 60, Pages 1422-1423, FIG. 60-3 and Table 60-2 c) Rationale: Attendant care 10 hr/day would be more appropriate for a patient with a C5 spinal cord injury. A patient with a L1 spinal cord injury would likely not require this much assistance. Chapter 60, Page 1423, Table 60-2 d) Rationale: Instability of lower back is characteristic of L1-2 spinal cord injuries. Patients may be able to regain good sitting balance through rehab. Chapter 60, Page 1423, Table 60-2 e) Rationale: Full innervation of upper extremities would be expected in a patient with a spinal cord injury below the T1 level. Chapter 60, Page 1423, Table 60-2

Which clinical manifestations are seen in the patient with hypertrophic CMP (Select all that applies) a) Dyspnea b) Angina c) Syncope d) Sydenham's chorea

a, b, c. Rationale: a) The most common symptom is dyspnea, which is caused by an elevated left ventricular diastolic pressure. Ch 36, p - 798. b) Angina is most often caused by the increased left ventricular mass or compression of the small coronary arteries by the hypertrophic ventricular myocardium. Ch 36, p - 798. c) The patient may have syncope, especially during exertion. Syncope is most often caused by an increase in obstruction to aortic outflow during increased activity. Ch 36, p - 799. d) Sydenham's chorea is the major CNS manifestation of RF, not CMP. Ch 36, p - 789.


Kaugnay na mga set ng pag-aaral

Chapter 14: Emotional and Social Development in Early Adulthood

View Set

AP Bio Final Review From DNA to Protein

View Set

CHAPTER 7 GOVERNMENT - The Mass Media and the Political Agenda

View Set

Compensation Administration - Chapter 10

View Set

Chapter 12: Reporting Cash Flows

View Set

Grade 5. SST. Beginning of the British Rule in India.

View Set